matheraum.de
Raum für Mathematik
Offene Informations- und Nachhilfegemeinschaft

Für Schüler, Studenten, Lehrer, Mathematik-Interessierte.
Hallo Gast!einloggen | registrieren ]
Startseite · Forum · Wissen · Kurse · Mitglieder · Team · Impressum
Forenbaum
^ Forenbaum
Status Mathe
  Status Schulmathe
    Status Primarstufe
    Status Mathe Klassen 5-7
    Status Mathe Klassen 8-10
    Status Oberstufenmathe
    Status Mathe-Wettbewerbe
    Status Sonstiges
  Status Hochschulmathe
    Status Uni-Analysis
    Status Uni-Lin. Algebra
    Status Algebra+Zahlentheo.
    Status Diskrete Mathematik
    Status Fachdidaktik
    Status Finanz+Versicherung
    Status Logik+Mengenlehre
    Status Numerik
    Status Uni-Stochastik
    Status Topologie+Geometrie
    Status Uni-Sonstiges
  Status Mathe-Vorkurse
    Status Organisatorisches
    Status Schule
    Status Universität
  Status Mathe-Software
    Status Derive
    Status DynaGeo
    Status FunkyPlot
    Status GeoGebra
    Status LaTeX
    Status Maple
    Status MathCad
    Status Mathematica
    Status Matlab
    Status Maxima
    Status MuPad
    Status Taschenrechner

Gezeigt werden alle Foren bis zur Tiefe 2

Navigation
 Startseite...
 Neuerdings beta neu
 Forum...
 vorwissen...
 vorkurse...
 Werkzeuge...
 Nachhilfevermittlung beta...
 Online-Spiele beta
 Suchen
 Verein...
 Impressum
Das Projekt
Server und Internetanbindung werden durch Spenden finanziert.
Organisiert wird das Projekt von unserem Koordinatorenteam.
Hunderte Mitglieder helfen ehrenamtlich in unseren moderierten Foren.
Anbieter der Seite ist der gemeinnützige Verein "Vorhilfe.de e.V.".
Partnerseiten
Dt. Schulen im Ausland: Mathe-Seiten:Weitere Fächer:

Open Source FunktionenplotterFunkyPlot: Kostenloser und quelloffener Funktionenplotter für Linux und andere Betriebssysteme
StartseiteMatheForenAlgebraische GeometrieArtinsch, idempotente Elemente
Foren für weitere Schulfächer findest Du auf www.vorhilfe.de z.B. Deutsch • Englisch • Französisch • Latein • Spanisch • Russisch • Griechisch
Forum "Algebraische Geometrie" - Artinsch, idempotente Elemente
Artinsch, idempotente Elemente < Algebraische Geometrie < Algebra < Algebra+Zahlentheo. < Hochschule < Mathe < Vorhilfe
Ansicht: [ geschachtelt ] | ^ Forum "Algebraische Geometrie"  | ^^ Alle Foren  | ^ Forenbaum  | Materialien

Artinsch, idempotente Elemente: Frage (beantwortet)
Status: (Frage) beantwortet Status 
Datum: 22:25 Di 13.11.2012
Autor: Teufel

Aufgabe
Sei A ein artinscher Ring mit mehr als 2 maximalen Idealen. Zeige: Es existieren idempotente Elemente außer 0 und 1.

Hi!

Ich weiß nicht, wie ich hier ansetzen kann. Ein idempotentes Element konstruieren, stelle ich mir hier schwierig vor. Also sollte ich hier vielleicht mit einem Widerspruchsbeweis argumentieren.

(Soll mit "mehr als 2 maximalen Idealen" vielleicht die zwei auch noch eingeschlossen sein? Würde meiner Meinung nach mehr Sinn ergeben.)

Ok, also ich weiß, dass A artinsch ist, d.h. absteigende Ketten von Idealen werden stationär. Annahme: 0 und 1 sind die einzigen idempotenten Elemente, also würde auch [mm] $e=e^2$ [/mm] stets $e=0$ oder $e=1$ folgen.
Ich nehme an, dass ich jetzt eine tolle fallende Idealkette erzeugen könnte. Und dabei muss ich wohl noch irgendwie verbraten, dass es mehr als 2 maximale Ideale gibt. Aber ich habe keine Ahnung, was ich da anstellen könnte, um einen Widerspruch zu erzeugen. Hat jemand einen Tipp parat?

Vielen Dank!


        
Bezug
Artinsch, idempotente Elemente: Antwort
Status: (Antwort) fertig Status 
Datum: 23:11 Di 13.11.2012
Autor: felixf

Moin!

> Sei A ein artinscher Ring mit mehr als 2 maximalen Idealen.
> Zeige: Es existieren idempotente Elemente außer 0 und 1.
>  
> Ich weiß nicht, wie ich hier ansetzen kann. Ein
> idempotentes Element konstruieren, stelle ich mir hier
> schwierig vor. Also sollte ich hier vielleicht mit einem
> Widerspruchsbeweis argumentieren.
>  
> (Soll mit "mehr als 2 maximalen Idealen" vielleicht die
> zwei auch noch eingeschlossen sein? Würde meiner Meinung
> nach mehr Sinn ergeben.)

Bei genau zwei maximalen Idealen gilt es bereits auch.

Hattet ihr schon die Aussage, dass artinsche Ringe das Produkt ihrer Lokalisierungen an den maximalen Idealen sind? Damit ist es recht einfach.

> Ok, also ich weiß, dass A artinsch ist, d.h. absteigende
> Ketten von Idealen werden stationär. Annahme: 0 und 1 sind
> die einzigen idempotenten Elemente, also würde auch [mm]e=e^2[/mm]
> stets [mm]e=0[/mm] oder [mm]e=1[/mm] folgen.
>  Ich nehme an, dass ich jetzt eine tolle fallende
> Idealkette erzeugen könnte. Und dabei muss ich wohl noch
> irgendwie verbraten, dass es mehr als 2 maximale Ideale
> gibt. Aber ich habe keine Ahnung, was ich da anstellen
> könnte, um einen Widerspruch zu erzeugen. Hat jemand einen
> Tipp parat?

Nimm doch ein Element $x [mm] \in [/mm] R [mm] \setminus R^\ast$, [/mm] welches nicht nilpotent ist (wenn alle Nicht-Einheiten nilpotent sind gibt es genau ein maximales Ideal -- was aber nicht der Fall ist nach Annahme). Betrachte nun die Ideale $(x), [mm] (x^2), (x^3), \dots$. [/mm] Dies ist eine absteigende Kette von Idealen, womit es ein $n [mm] \in \IN$ [/mm] gibt mit [mm] $(x^n) [/mm] = [mm] (x^{n+1})$. [/mm] Damit ist [mm] $x^n [/mm] = a [mm] \cdot x^{n+1}$ [/mm] fuer ein $a [mm] \in [/mm] R$.

Damit kannst du dir jetzt ein nicht-triviales idempotentes Element konstruieren.

LG Felix


Bezug
                
Bezug
Artinsch, idempotente Elemente: Frage (beantwortet)
Status: (Frage) beantwortet Status 
Datum: 00:11 Mi 14.11.2012
Autor: Teufel

Hi nochmal!

> Hattet ihr schon die Aussage, dass artinsche Ringe das
> Produkt ihrer Lokalisierungen an den maximalen Idealen
> sind? Damit ist es recht einfach.
>  

Nein, leider nicht.

>  
> Nimm doch ein Element [mm]x \in R \setminus R^\ast[/mm], welches
> nicht nilpotent ist (wenn alle Nicht-Einheiten nilpotent
> sind gibt es genau ein maximales Ideal -- was aber nicht
> der Fall ist nach Annahme).

Wieso gilt das denn? Den Satz hatten wir leider auch nicht. Ist das Nilradikal dann das eindeutige maximale Ideal?

> Betrachte nun die Ideale [mm](x), (x^2), (x^3), \dots[/mm].
> Dies ist eine absteigende Kette von Idealen, womit es ein [mm]n \in \IN[/mm]
> gibt mit [mm](x^n) = (x^{n+1})[/mm]. Damit ist [mm]x^n = a \cdot x^{n+1}[/mm]
> fuer ein [mm]a \in R[/mm].
>  

Ok.

> Damit kannst du dir jetzt ein nicht-triviales idempotentes
> Element konstruieren.
>  

Ok, ich schau mal, ob ich das hinbekomme. Gerade fällt mir nichts mehr ein, an der das liegt wohl an der Uhrzeit.

> LG Felix
>  


Vielen Dank für deine Hilfe!

Bezug
                        
Bezug
Artinsch, idempotente Elemente: Antwort
Status: (Antwort) fertig Status 
Datum: 14:23 Mi 14.11.2012
Autor: Schadowmaster


> > Nimm doch ein Element [mm]x \in R \setminus R^\ast[/mm], welches
> > nicht nilpotent ist (wenn alle Nicht-Einheiten nilpotent
> > sind gibt es genau ein maximales Ideal -- was aber nicht
> > der Fall ist nach Annahme).
>  Wieso gilt das denn? Den Satz hatten wir leider auch
> nicht. Ist das Nilradikal dann das eindeutige maximale
> Ideal?

Genau.
Dass ein Ring, in dem die Menge der Nichteinheiten ein Ideal bilden (also in diesem Fall das Nilradikal) genau ein maximales Ideal besitzt (nämlich genau die Menge der Nichteinheiten) kannst du dir sicher ganz einfach klar machen.

Bezug
                                
Bezug
Artinsch, idempotente Elemente: Mitteilung
Status: (Mitteilung) Reaktion unnötig Status 
Datum: 15:56 Mi 14.11.2012
Autor: Teufel

Ah ok, vielen Dank!

Ich habe für eine andere Aufgabe auch gerade schauen müssen, was ein lokaler Ring sein soll. Dort habe ich nun auch diese Aussage gefunden (Falls R kommutativ: [mm] R\setminus R^\times [/mm] Ideal [mm] \gdw [/mm] R hat genau ein max. Ideal).

Schlimm, wenn man die Begriffe und ihre Äquivalenzen dazu nicht kennt. Na dann mach ich mich mal wieder an die Aufgabe!

Bezug
                
Bezug
Artinsch, idempotente Elemente: Frage (beantwortet)
Status: (Frage) beantwortet Status 
Datum: 17:06 Mi 14.11.2012
Autor: Teufel

Hi nochmal!

Ich sitze nun schon eine Weile daran und komme irgendwie nicht auf die Konstruktion. Ich weiß Folgendes:

x und [mm] x^n [/mm] (und alle größeren Potenzen von x) sind keine Einheiten und auch nicht nilpotent. Inbesondere sind all diese Elemente ungleich 0 oder 1. Daher würde es sich natürlich anbieten, wenn das idempotente Elemente irgendeine x-Potenz sein könnte.

Aber irgendwie komme ich nicht vorwärts. Ich wollte dann versuchen das a noch mit reinzubringen, aber wenn ich z.B. a*x rechne, dann könnte das ja schon wieder ein Nullteiler oder eine Einheit sein. Ich wollte solche Sachen machen wie [mm] x^n=a*x^{n+1} \Rightarrow (ax)^n=(ax)^{n+1}. [/mm]
Dann habe ich mir Sachen angeschaut wie [mm] x^n=b*x^{2n}, [/mm] aber hier stört mich das b (wäre es nicht da, hätte ich [mm] x^n [/mm] als idempotentes Element ausgemacht).

Irgendwie sehe ich gerade nicht, was man da machen kann.

Bezug
                        
Bezug
Artinsch, idempotente Elemente: Antwort
Status: (Antwort) fertig Status 
Datum: 10:29 Do 15.11.2012
Autor: felixf

Moin!

> x und [mm]x^n[/mm] (und alle größeren Potenzen von x) sind keine
> Einheiten und auch nicht nilpotent. Inbesondere sind all
> diese Elemente ungleich 0 oder 1. Daher würde es sich
> natürlich anbieten, wenn das idempotente Elemente
> irgendeine x-Potenz sein könnte.

Nicht direkt.

> Aber irgendwie komme ich nicht vorwärts. Ich wollte dann
> versuchen das a noch mit reinzubringen, aber wenn ich z.B.
> a*x rechne, dann könnte das ja schon wieder ein Nullteiler
> oder eine Einheit sein. Ich wollte solche Sachen machen wie
> [mm]x^n=a*x^{n+1} \Rightarrow (ax)^n=(ax)^{n+1}.[/mm]

Wendest du das ein paarmal oefter an, kannst du damit $((a [mm] x)^n)^2 [/mm] = (a [mm] x)^n$ [/mm] zeigen.

LG Felix


Bezug
                                
Bezug
Artinsch, idempotente Elemente: Mitteilung
Status: (Mitteilung) Reaktion unnötig Status 
Datum: 16:48 Do 15.11.2012
Autor: Teufel

Puh, da stand ich echt mal wieder auf dem Schlauch. Wie machst du das immer nur, dass du einfach alles weißt? :)

Ok, also ich habe es jetzt mit deiner Hilfe hinbekommen. Ich konnte auch noch zeigen, dass $ax$ weder Einheit noch nilpotent ist.

Vielen Dank noch einmal!

Bezug
Ansicht: [ geschachtelt ] | ^ Forum "Algebraische Geometrie"  | ^^ Alle Foren  | ^ Forenbaum  | Materialien


^ Seitenanfang ^
www.matheraum.de
[ Startseite | Forum | Wissen | Kurse | Mitglieder | Team | Impressum ]